Proving the Existence of a Maximum or Minimum for a Continuous Function

  • Thread starter Thread starter MathematicalPhysicist
  • Start date Start date
  • Tags Tags
    Continuity
Click For Summary
To prove that a continuous function f: R -> R attains a maximum or minimum given that lim f(x) = L as x approaches both positive and negative infinity, the Weierstrass theorem is essential. The discussion highlights that within defined intervals, the function achieves both maximum and minimum values. The approach suggests examining the supremum of the function; if it is finite and greater than L, one can demonstrate that the function must attain this supremum as a maximum. Conversely, if the function does not have a maximum, it implies that the maxima in successive intervals are increasing, which contradicts the assumption of finiteness. Ultimately, the argument centers on the continuity of the function and the behavior of its limits at infinity.
MathematicalPhysicist
Science Advisor
Gold Member
Messages
4,662
Reaction score
372
i have a continuous function f:R->R and we are given that lim f(x)=L as x approaches infinity and limf(x)=L as x approaches minus infinity, i need to prove that f gets a maximum or minimum in R.

obviously i need to use weirstrauss theorem, but how to implement it in here.
i mean by defintion:
Ae>0,EM_e,Ax>=M_e, |f(x)-L|<e
Ae>0,Em_e,Ax<=m_e,|f(x)-L|<e

so if we look at the intervals:
[M1,M2],[M1,M3]...
[m2,m1],[m3,m1]...
in each interval the function gets a maximum and minimum by the theorem i quoted above, at the capital M's as x ais bigger than M1 its interval of f is increased i.e for x>=M1 L-1<f(x)<L+1 for x >=M2 L-2<f(x)<L+2 so it means the maximum in the first interval isn't bigger than the maximum in the second interval (the problem is i cannot say the same about the minimum).
now if f has a maximum then we finished if it doesn't then i should show it has a minimum, but if it doesn't have a maximum then each maximum in the intervals is bigger than the previous one, but we have that there isn't a maximum.
here I am stuck and i don't know how to procceed from here, any help will be appreciated.
 
Physics news on Phys.org
I don't understand your method. Why would it treat maxima any different than minima?

Here's how I would do it. You can always take the sup of a function. Show the sup of this function is finite. If it's greater than L, find a way to show the function must actually attain this value, making it a max. You can go from here.
 
Last edited:
Question: A clock's minute hand has length 4 and its hour hand has length 3. What is the distance between the tips at the moment when it is increasing most rapidly?(Putnam Exam Question) Answer: Making assumption that both the hands moves at constant angular velocities, the answer is ## \sqrt{7} .## But don't you think this assumption is somewhat doubtful and wrong?

Similar threads

Replies
30
Views
3K
  • · Replies 11 ·
Replies
11
Views
2K
  • · Replies 3 ·
Replies
3
Views
2K
  • · Replies 8 ·
Replies
8
Views
2K
  • · Replies 2 ·
Replies
2
Views
1K
Replies
5
Views
2K
  • · Replies 11 ·
Replies
11
Views
2K
  • · Replies 5 ·
Replies
5
Views
2K
  • · Replies 2 ·
Replies
2
Views
2K
Replies
7
Views
1K